Jump to content

analysis90's Content

There have been 38 items by analysis90 (Search limited from 02-06-2020)



Sort by                Order  

#306948 Đề thi ôn tập thường xuyên của ĐHĐT

Posted by analysis90 on 30-03-2012 - 09:46 in Thảo luận về các kì thi, các kì kiểm tra Toán sinh viên

MATHEMATICAL OLYMPIAD STUDENT

(third-2012)

Exercise 1. For $f(x)=2(x-1)-\arctan x,x\in\mathbb{R}$.
a) Prove that $f(x)=0$ have only a root $a\in(1,\sqrt{3})$.
b) Let $\{u_{n}\}_{n=1}^{\infty}$ be a sequence defined by$\left\{\begin{matrix} u_1=\dfrac{3}{2}& \\ u_n=1+\dfrac{1}{2}\arctan x,&n\geq1 \end{matrix}\right.$.
Prove that $\{u_{n}\}_{n=1}^{\infty}$ converges to $a$.
Exercise 2. Let $f:[0,+\infty)\longrightarrow \mathbb{R}$ be a differentiable function such that $f(0)=1$. Prove that if $f'(x)\geq f(x)$ for all $x\in[0,+\infty)$, then the function $g(x)=f(x)-e^x$ is a increasing function.
Exercise 3. Let $f:[0,1]\longrightarrow \mathbb{R}$ be a integrable function such that $\int_0^1xf(x)dx=0$. Prove that
$\int_0^1f^2(x)dx\geq 4(\int_0^1f(x)dx)^2$.
Exercise 4. Let $f:\mathbb{R}\longrightarrow \mathbb{R}$ be a twice differentiable, $g:\mathbb{R}\longrightarrow \mathbb{R}^+$ be a function such that $f(x)+f''(x)=-xg(x)f'(x)$ for all $x\in\mathbb{R}$. Prove that $f(x)$ is bounded.
Exercise 5. Let $P(x)=\sum_{i=0}^{n}a_ix^i$ with $a_n>0$ be a $n$ degrees polynomial and have distinct $n$ roots. Prove that the polynomial $Q(x)=(P(x))^2-P'(x)$ only have
a) distinct $n+1$ roots if $n$ is odd.
b) distinct $n$ roots if $n$ is even.
Exercise 6. Find al function $f:\mathbb{R}\longrightarrow \mathbb{R}$ satisfies
$f(x+y)\geq f(x).f(y)\geq e^{x+y}$ for all $x,y\in \mathbb{R}$.



#307504 Đề thi ôn tập thường xuyên của ĐHĐT

Posted by analysis90 on 01-04-2012 - 12:13 in Thảo luận về các kì thi, các kì kiểm tra Toán sinh viên

Exercise 3. We have $\int_0^1f^2(x)dx\int_0^2(3x-2)^2dx\geq (\int_0^1f(x)(3x-2)dx)^2$.



#299744 Đề thi Olympic toán sinh viên cấp trường của Đại học kinh tế quốc dân năm 2012

Posted by analysis90 on 17-02-2012 - 15:44 in Thảo luận về các kì thi, các kì kiểm tra Toán sinh viên

Assumming $g(x)=ax+b$ such that $\int_0^1g(x)dx=\int_0^1xg(x)=1$. Hence, $g(x)=6x-2$.
We have $\int_0^1(f(x)-6x+2)^2dx\geq 0$, but $\int_0^1(f(x)-6x+2)^2dx=\int_0^1(f(x))^2dx-4$



#302006 Đề thi Olympic toán sinh viên cấp trường của Đại học kinh tế quốc dân năm 2012

Posted by analysis90 on 03-03-2012 - 13:16 in Thảo luận về các kì thi, các kì kiểm tra Toán sinh viên

Problem 2. We have $g(0)=0$ and $g(x)$ be a continuous. Integral both sides, we get
$\int_0^xg(y)dy=(\int_0^yf(t)dt)^{2012}\geq 0$.
Assign $h(x)=\int_0^xg(t)dt$. So, we have $h'(x)=g(x)$.
Because $g(x)$ be a nonincreasing, so $g(x)\leq g(0),\forall x\in[0,+\infty)$. Therefore
$h'(x)\leq 0,\forall x\in[0,+\infty)$ or $h(x)\leq h(0)=0,\forall x\in[0,+\infty)$
Implies
$h(x)=0,\forall x\in[0,+\infty)$.
Similarly for $x\in(-\infty,0]$. We have $h(x)=0$ or $g(x)=0,\forall x\in \mathbb{R}$.
When $\int_0^xf(t)dt=0,\forall x\in\mathbb{R}$. Easily, we prove that $f(x)=0$



#302224 Đề thi chọn đội tuyển Olympic SV 2012 môn Giải tích - ĐHKHTN, ĐHQGHN

Posted by analysis90 on 04-03-2012 - 20:38 in Thảo luận về các kì thi, các kì kiểm tra Toán sinh viên

problem 3. For all $\epsilon>0,\exists n_0$ such that $|2a_{n+1}-a_n-2012|<\epsilon,\forall n\geq n_0$. Hence,
$|2(a_{n+1}-2012)-(a_n-2012)|<\epsilon$
Implies
$|a_{n+1}-2012|<\dfrac{\epsilon}{2}+\dfrac{1}{2}|a_n-2012| <\dots<\epsilon(\dfrac{1}{2}+\dfrac{1}{4}+\dots+\dfrac{1}{2^{n-n_0}})=\epsilon(1-(\dfrac{1}{2} )^{n-n_0})<\epsilon$
We have $\lim a_n=2012$



#302253 Đề thi chọn đội tuyển Olympic SV 2012 môn Giải tích - ĐHKHTN, ĐHQGHN

Posted by analysis90 on 04-03-2012 - 22:03 in Thảo luận về các kì thi, các kì kiểm tra Toán sinh viên

Problem 2. Every $x\in[0,2]$, we have
$f(x)=f(x)-f(0)=f'(\theta_1)x,\theta_1\in(0,x)\geq-2x$
$f(x)-f(1)=f'(\theta_2)(x-1),\theta_2\in(x,1)\Rightarrow f(x)\geq 2x-1$.
So,
$\int_0^1f(x)dx=\int_0^\frac{1}{4}f(x)dx+\int_\frac{1}{4}^1f(x)dx
\geq \int_0^\frac{1}{4}-2xdx+\int_\frac{1}{4}^1 (2x-1)dx=\dfrac{1}{8}$
But $f(x)=\left\{\begin{matrix}
-2x &x\in[0,\frac{1}{4}] \\
2x-1&x\in[\frac{1}{4},1]
\end{matrix}\right.$
isn't continuous at $x=\dfrac{1}{4} $.



#299901 Đề thi Olympic toán học sinh viên 2012 Đại Học BK Hà Nội

Posted by analysis90 on 18-02-2012 - 22:13 in Thảo luận về các kì thi, các kì kiểm tra Toán sinh viên

Exercise 5. We have $f(t)=\int_0^1f(x)dx+\int_0^txf'(x)dx+\int_t^1(x-1)f'(x)dx$, forall $t\in [0,1]$.
Therefore $|f(t)|\leq \int_0^1|f(x)|dx+t\int_0^t|f'(x)|dx+(1-t)\int_t^1|f'(x)|dx$.
Final, we choose $t=\dfrac{1}{2} $.



#299639 Ôn thi Olympic Toán học sinh viên 2015 [Giải tích]

Posted by analysis90 on 16-02-2012 - 16:06 in Thảo luận về các kì thi, các kì kiểm tra Toán sinh viên

Exercise 4: Find all integrable function $f:[0,1]\rightarrow \mathbb{R}$ such that
$f(x)\leq \int_0^x t^{2012}f(t)dt , \forall x \in [0,1]$.



#303046 Ôn thi Olympic Toán học sinh viên 2015 [Giải tích]

Posted by analysis90 on 09-03-2012 - 08:25 in Thảo luận về các kì thi, các kì kiểm tra Toán sinh viên

Problem 5. Assign $g(x)=e^x[f(x)-1]+1$. We have $g'(x)=e^x[f(x)-1+f'(x)]<0$. So,
$e[f(1)-1]+1=g(1)<g(0)=0$
If $f(x)+f'(x)<1$ then there don't exist $f(x)$.



#299546 Ôn thi Olympic Toán học sinh viên 2015 [Giải tích]

Posted by analysis90 on 15-02-2012 - 21:53 in Thảo luận về các kì thi, các kì kiểm tra Toán sinh viên

Consider $F(x)=\dfrac{1}{x} \int_0^{x}f(t)dt$. We have $f(x)=(xF(x))'$ and
$\int_0^{1}f(x)dx=(xF(x))|_0^{1}=F(1)$
$2\int_{\dfrac{1}{4}}^{\dfrac{3}{4}}f(x)dx=\dfrac{3}{2}F(\dfrac{3}{4})-\dfrac{1}{2}F(\dfrac{1}{4}) $.
Implies
$3F(\dfrac{3}{4} )-F(\dfrac{1}{4} )=2F(1)$
$\Leftrightarrow F(\dfrac{3}{4})-F(\dfrac{1}{4})=2(F(1)-F(\dfrac{3}{4}))$
So, there exist $\theta_{1}\in (\dfrac{1}{4},\dfrac{3}{4})$ and $\theta_{2}\in (\dfrac{3}{4},1)$ such that
$F'(\theta_1)\dfrac{1}{2}=2.F'(\theta_2)\dfrac{1}{4}$
$\Rightarrow F'(\theta_1)= F'(\theta_2)$
we have $\theta\in (\theta_1,\theta_2)$ such that $F''(\theta)=0$ or
$\dfrac{2}{\theta^3} \int_0^{\theta}f(t)dt-\dfrac{2}{\theta^2}f(\theta)+\dfrac{1}{\theta}f'(\theta)=0 $
Next, assignment $H(x)=2\int_0^{x}f(t)dt-2xf(x)+x^2f'(x)$
$\Rightarrow H(0)=0, H(\theta)=0$
$\exists x_0\in (0,\theta):H'(x_0)=0$
But $H'(x)=x^2f''(x)$
Final $f''(x_0)=0$



#300784 [Thắc mắc] Về kỳ thi Olympic Toán Sinh Viên

Posted by analysis90 on 24-02-2012 - 19:00 in Thảo luận về các kì thi, các kì kiểm tra Toán sinh viên

Bạn phải hiểu đây là sân chơi để khẳng định mình, khẳng định trường mình. Còn được gì, thì ngoài vật chất ''những tấm bằng khen" (rất khó để có) và tiền, thì cái bạn có nữa là cơ hội giao lưu bạn bè về toán học. Đề đại số những năm gần không "khó" lắm. Với theo tôi cái khó của người này đôi khi là cái dễ của bản thân nữa. Thân chào bạn.



#240555 1 bài toán không hề dễ

Posted by analysis90 on 11-09-2010 - 21:05 in Bất đẳng thức và cực trị

nếu hệ là nhừ thế này $\left\{\begin{array}{l}x^3+y=2\\y^3+x=2\end{array}\right. $ thì có thể giải như sau;
trừ hai vế pt ta có pt $(x-y)(x^2+y^2+xy-1)=0\Leftrightarrow$ $x=y$ hoặc $x^2+y^2+xy-1=0$
với x=y thì $x^3+x-2=0 \Leftrightarrow (x-1)(x^2+x+2)=0$
với $X^2+y^2+xy-1=0$ thì bó tay



#237729 1 bài toán chưa rõ

Posted by analysis90 on 21-08-2010 - 08:35 in Thi tốt nghiệp

đề đúng rồi